Why Does \( e^{\frac{15i\pi}{2}} \) Simplify to \( e^{\frac{3i\pi}{2}} \)?

Click For Summary
The discussion centers on simplifying the expression \( e^{\frac{15i\pi}{2}} \) to \( e^{\frac{3i\pi}{2}} \) by recognizing that angles can be reduced within the range of \( 0 \) to \( 2\pi \). The calculation for \( z^10 \) where \( z = -1 + i \) leads to \( z^{10} = 32e^{i(15\pi/2)} \), which simplifies correctly to \( 32e^{i(3\pi/2)} \). Participants clarify that both the book's answer and the calculated result are technically correct, but the book's format adheres to standard principal argument conventions. The importance of keeping angles within specified ranges is emphasized for clarity in complex number representations. Understanding these simplifications is crucial for accurate mathematical communication.
Phyisab****
Messages
584
Reaction score
2

Homework Statement



Obtain z^10 for z=-1+i


Homework Equations



z=re^i(theta)


The Attempt at a Solution



Theta is 3pi/4. So z^10 = 32e^i(15pi/2).

The answer in my book is 32e^i(3pi/2) but I'm pretty sure that's wrong, can anyone confirm?
 
Physics news on Phys.org
yes but e^{\frac{15 i \pi}{2}} = e^{\frac{12 i \pi}{2}} \cdot e^{\frac{3 i \pi}{2}}

does that help?
 
z = -1 + i = sqrt(2) cis(3/4 pi)

z^10 = 2^5 cis (10 x 3/4 pi) {De Moivre's Theorem}
= 32 cis (15/2 pi)
= -32i
 
So I was right? Why did the answer appear in the book that way? Or is it wrong?
 
Both the book and your answer is right -
though technically the answers should be 32e^-i(1/2pi).

remember, the range of principal argument is 0 < theta < pi (above the x-axis) &
-pi < theta < 0 (below the x-axis). so whenever you get a value beyond these ranges you have to convert your large value into an equivalent but smaller value within the range.

So you start at the line to the right of the origin of the x-axis (which is theta = 0). one revolution around all the quadrants back to the start is 2 pi. you keep on going until you finish up to 15/2 pi. you should end up at the line to the bottom of the origin of the y-axis. that is 3/2 pi.
 
latentcorpse said:
yes but e^{\frac{15 i \pi}{2}} = e^{\frac{12 i \pi}{2}} \cdot e^{\frac{3 i \pi}{2}}

does that help?

Phyisab**** said:
So I was right? Why did the answer appear in the book that way? Or is it wrong?
The point is that 12 i \pi= 6 (2 i \pi) so that e^{12 i\pi}= (e^{2 i\pi})^6= 1.

e^{15 i \pi}= e^{3 i\pi} but it is best always to write the argument between 0 and 2\pi.
 
Question: A clock's minute hand has length 4 and its hour hand has length 3. What is the distance between the tips at the moment when it is increasing most rapidly?(Putnam Exam Question) Answer: Making assumption that both the hands moves at constant angular velocities, the answer is ## \sqrt{7} .## But don't you think this assumption is somewhat doubtful and wrong?

Similar threads

  • · Replies 11 ·
Replies
11
Views
1K
Replies
7
Views
2K
Replies
32
Views
3K
  • · Replies 7 ·
Replies
7
Views
2K
  • · Replies 7 ·
Replies
7
Views
3K
Replies
9
Views
2K
  • · Replies 22 ·
Replies
22
Views
902
  • · Replies 9 ·
Replies
9
Views
5K
  • · Replies 1 ·
Replies
1
Views
2K
  • · Replies 4 ·
Replies
4
Views
2K